Convolution of gaussian functions

Click For Summary
The discussion revolves around the convolution of Gaussian functions defined as ##f_s(t) = \sqrt{s}e^{-st^2}##. The problem requires demonstrating that ##f_3 \ast f_6(t) = \sqrt{\pi}f_{1/2}(t) = \sqrt{\pi/2}e^{-t^{2}/2}##. Participants express confusion over their calculations, particularly encountering an imaginary unit in their results and discrepancies in the expected answers. Suggestions include using the Fourier transform method and correcting an error in the exponent during integration. A teacher's correction indicates that the actual result should be ##f_3 \ast f_6 = \sqrt{\pi} f_2(t) = \sqrt{2\pi}e^{-2t^2}##, highlighting the importance of normalization factors in the transformation process.
ElijahRockers
Gold Member
Messages
260
Reaction score
10

Homework Statement


Recall that we have defined the Gaussian ##f_s## by ##f_s (t) = \sqrt{s}e^{-st^2}## and shown that ##\hat{f_s}(\lambda) = \frac{1}{\sqrt{2}}e^{\frac{-\lambda^2}{4s}}##.

Show that ##f_3 \ast f_6 (t) = \sqrt{\pi}f_{1/2}(t) = \sqrt{\pi/2}e^{-t^{2}/2}##

The Attempt at a Solution



Not sure what's wrong with my approach, but I'm getting ##i## in both of my attempts answers, and besides that my answers are no where near close to the correct answer. Each of the pages represents a single attempt. I first tried multiplying the Fourier transforms of both functions then taking the inverse, and when that didn't work, I tried using the definition of convolution.

Image is attached but resized is hard to read... full size is here
 

Attachments

  • 20150324_221826.jpg
    20150324_221826.jpg
    50.6 KB · Views: 535
Physics news on Phys.org
ElijahRockers said:

Homework Statement


Recall that we have defined the Gaussian ##f_s## by ##f_s (t) = \sqrt{s}e^{-st^2}## and shown that ##\hat{f_s}(\lambda) = \frac{1}{\sqrt{2}}e^{\frac{-\lambda^2}{4s}}##.

Show that ##f_3 \ast f_6 (t) = \sqrt{\pi}f_{1/2}(t) = \sqrt{\pi/2}e^{-t^{2}/2}##

The Attempt at a Solution



Not sure what's wrong with my approach, but I'm getting ##i## in both of my attempts answers, and besides that my answers are no where near close to the correct answer. Each of the pages represents a single attempt. I first tried multiplying the Fourier transforms of both functions then taking the inverse, and when that didn't work, I tried using the definition of convolution.

Image is attached but resized is hard to read... full size is here
On the left hand page, you're fine down to here:
Capture_ERockers_1.PNG

The line you have after is in error.
It would have been better pull out only ##\displaystyle\ e^{-2t^2} \,,\,## leaving an integrand of ##\displaystyle\ e^{\displaystyle-(9\tau^2-12\tau\,t+4t^2)} \ ## .

The exponent is the negative of a perfect square. Use a substitution to solve the integral.

On the right hand page, you have the following:
Capture_ERockers_2.PNG

Therefore, simply multiply ##\displaystyle\ \hat{f_3}(\lambda) \hat{f_6}(\lambda) \ ## . Then simply transform back using:
##\hat{f_s}(\lambda) = \frac{1}{\sqrt{2}}e^{\frac{-\lambda^2}{4s}}##​
in reverse.
 
  • Like
Likes ElijahRockers
Hmmm... transforming back using the inverse Gaussian formula should be rather straightforward, but I don't see where the pi comes from in the final answer.

EDIT: Also the teacher has made a correction to the problem, the answer should be ##f_3 \ast f_6 = \sqrt{\pi} f_2 (t) = \sqrt{\pi/2}e^{-2t^2}## which still doesn't make sense, but I'm inclined to believe he meant ##\sqrt{\pi} f_2 (t) = \sqrt{2\pi}e^{-2t^2}##
 
ElijahRockers said:
Hmmm... transforming back using the inverse Gaussian formula should be rather straightforward, but I don't see where the pi comes from in the final answer.

EDIT: Also the teacher has made a correction to the problem, the answer should be ##f_3 \ast f_6 = \sqrt{\pi} f_2 (t) = \sqrt{\pi/2}e^{-2t^2}## which still doesn't make sense, but I'm inclined to believe he meant ##\sqrt{\pi} f_2 (t) = \sqrt{2\pi}e^{-2t^2}##
I forgot, transforming back, there is a normalization factor to consider.
 
  • Like
Likes ElijahRockers
Question: A clock's minute hand has length 4 and its hour hand has length 3. What is the distance between the tips at the moment when it is increasing most rapidly?(Putnam Exam Question) Answer: Making assumption that both the hands moves at constant angular velocities, the answer is ## \sqrt{7} .## But don't you think this assumption is somewhat doubtful and wrong?

Similar threads

  • · Replies 1 ·
Replies
1
Views
2K
  • · Replies 13 ·
Replies
13
Views
2K
  • · Replies 1 ·
Replies
1
Views
1K
  • · Replies 1 ·
Replies
1
Views
2K
  • · Replies 1 ·
Replies
1
Views
1K
  • · Replies 6 ·
Replies
6
Views
3K
  • · Replies 47 ·
2
Replies
47
Views
4K
  • · Replies 6 ·
Replies
6
Views
1K
  • · Replies 9 ·
Replies
9
Views
3K
  • · Replies 1 ·
Replies
1
Views
1K